Hot News:

Mit Unterstützung durch:

  Foren auf CAD.de (alle Foren)
  ANSYS
  Elektrostatik

Antwort erstellen  Neues Thema erstellen
CAD.de Login | Logout | Profil | Profil bearbeiten | Registrieren | Voreinstellungen | Hilfe | Suchen

Anzeige:

Darstellung des Themas zum Ausdrucken. Bitte dann die Druckfunktion des Browsers verwenden. | Suche nach Beiträgen nächster neuer Beitrag | nächster älterer Beitrag
  
Gut zu wissen: Hilfreiche Tipps und Tricks aus der Praxis prägnant, und auf den Punkt gebracht für Ansys
Autor Thema:  Elektrostatik (2377 mal gelesen)
nohlesepiet
Mitglied



Sehen Sie sich das Profil von nohlesepiet an!   Senden Sie eine Private Message an nohlesepiet  Schreiben Sie einen Gästebucheintrag für nohlesepiet

Beiträge: 18
Registriert: 29.09.2011

erstellt am: 16. Nov. 2011 11:00    Editieren oder löschen Sie diesen Beitrag!  <-- editieren / zitieren -->   Antwort mit Zitat in Fett Antwort mit kursivem Zitat    Unities abgeben: 1 Unity (wenig hilfreich, aber dennoch)2 Unities3 Unities4 Unities5 Unities6 Unities7 Unities8 Unities9 Unities10 Unities

Morgen zusammen,

ich hab mal eine prinzipielle Frage an die E-Techniker:
Ich hab bisher nur die mechanische/ strukturdynamische Komponente von ANSYS genutzt.
Jetzt würde ich gerne in die Elektrostatik /dynamik. Ich würde gerne folgendes Problem lösen:
Elektisch geladene Teilchen, die sich in einem Spannungsfeld, also beispielsweise zwischen zwei Kondensatorplatten ins Gleichgewicht iterieren. Mir schwebt das ganze so vor, dass ich die Elektronen als masselose Punktelemente an einem beliebigen Ort in dem elektrischen Feld (wahlweise geht auch punktmagnete im magnetfeld, dass is mir im prinzip wurst) platziere und diese sich dann ihre gleichgewichtspositionen finden.
Ich weiß ich bin noch in dem mechanical denken aber sowas ähnliches wie z.b. mass21 elemente als elektrische quellen/punktmagnete die in ein viereck und auf die linien gebe ich eine Feldstärke vor. So oder so ähnlich....
Geht das in ANSYS?

Vielen Dank!

Eine Antwort auf diesen Beitrag verfassen (mit Zitat/Zitat des Beitrags) IP

nohlesepiet
Mitglied



Sehen Sie sich das Profil von nohlesepiet an!   Senden Sie eine Private Message an nohlesepiet  Schreiben Sie einen Gästebucheintrag für nohlesepiet

Beiträge: 18
Registriert: 29.09.2011

erstellt am: 16. Nov. 2011 16:06    Editieren oder löschen Sie diesen Beitrag!  <-- editieren / zitieren -->   Antwort mit Zitat in Fett Antwort mit kursivem Zitat    Unities abgeben: 1 Unity (wenig hilfreich, aber dennoch)2 Unities3 Unities4 Unities5 Unities6 Unities7 Unities8 Unities9 Unities10 Unities

Keiner ne Idee wie man sowas angeht??

Eine Antwort auf diesen Beitrag verfassen (mit Zitat/Zitat des Beitrags) IP

wosch
Ehrenmitglied V.I.P. h.c.
Elektrotechniker im Ruhestand


Sehen Sie sich das Profil von wosch an!   Senden Sie eine Private Message an wosch  Schreiben Sie einen Gästebucheintrag für wosch

Beiträge: 2607
Registriert: 16.12.2004

Rechne zuerst ein Problem nach, für das Du eine analytische Lösung kennst.

erstellt am: 16. Nov. 2011 17:57    Editieren oder löschen Sie diesen Beitrag!  <-- editieren / zitieren -->   Antwort mit Zitat in Fett Antwort mit kursivem Zitat    Unities abgeben: 1 Unity (wenig hilfreich, aber dennoch)2 Unities3 Unities4 Unities5 Unities6 Unities7 Unities8 Unities9 Unities10 Unities Nur für nohlesepiet 10 Unities + Antwort hilfreich

Nach meinem Verständnis bewegen sich elektrisch geladene Teilchen in einem elektrostatischen Feld, weil auf sie eine Kraft ausgeübt wird. F=Q*E
Ein Beispiel dazu findet sich in "FEM für Praktiker Band 4 Elektrotechnik".
Die hier angesprochene Aufgabenstellung habe ich allerdings nicht verstanden.

------------------
Viel Erfolg wünscht
Wolfgang Schätzing

Eine Antwort auf diesen Beitrag verfassen (mit Zitat/Zitat des Beitrags) IP

nohlesepiet
Mitglied



Sehen Sie sich das Profil von nohlesepiet an!   Senden Sie eine Private Message an nohlesepiet  Schreiben Sie einen Gästebucheintrag für nohlesepiet

Beiträge: 18
Registriert: 29.09.2011

erstellt am: 16. Nov. 2011 19:43    Editieren oder löschen Sie diesen Beitrag!  <-- editieren / zitieren -->   Antwort mit Zitat in Fett Antwort mit kursivem Zitat    Unities abgeben: 1 Unity (wenig hilfreich, aber dennoch)2 Unities3 Unities4 Unities5 Unities6 Unities7 Unities8 Unities9 Unities10 Unities


Teilchenproblem.png

 
Okay dann sind wir uns ja schon mal einig warum sich die Teilchen bewegen, dass entspricht auch meinem Verständnis.
Das Buch habe ich leider nicht. Zur Aufgabenstellung/Problem:
Ich platziere eine beliebige Anzahl geladene Teilchen (alle mit der gleichem Vorzeichen jedoch unterschiedlicher Ladung) in einer repräsentativen Zelle (2D) der Kantenlänge a.
Ich bin an einem periodischen Problem interessiert, d.h. ich simuliere einen Ausschnitt aus einem unendlichen 2D Raum. Ich habe zur Klarheit mal eine Prinzipskizze angehängt. Dabei stehen die unterschiedlichen Farben für unterschiedliche Ladungen.
Ich möchte an einem Zeitpunkt t=0 eine beliebige Position innerhalb meiner Zelle vorgeben, durch die Abstoßenden Kräfte der Teilchen entstehen Kräfte zwischen den Teilchen die für t-> unendlich zu einem Gleichgewichtszustand führen.  Ich habe das ganze für 1D in excel implementiert das ist etwas sperrig ich denke in ANSYS sollte das eleganter gehen.
Die Frage ist kann ich in Ansys eine solche (ich weiß nicht wie die korrekte Terminologie für das Problem ist ich bin Maschinenbauer) elektrostatische/dynamische lösen kann und wie das ganze anzugehen ist.

Eine Antwort auf diesen Beitrag verfassen (mit Zitat/Zitat des Beitrags) IP

wosch
Ehrenmitglied V.I.P. h.c.
Elektrotechniker im Ruhestand


Sehen Sie sich das Profil von wosch an!   Senden Sie eine Private Message an wosch  Schreiben Sie einen Gästebucheintrag für wosch

Beiträge: 2607
Registriert: 16.12.2004

Rechne zuerst ein Problem nach, für das Du eine analytische Lösung kennst.

erstellt am: 17. Nov. 2011 08:46    Editieren oder löschen Sie diesen Beitrag!  <-- editieren / zitieren -->   Antwort mit Zitat in Fett Antwort mit kursivem Zitat    Unities abgeben: 1 Unity (wenig hilfreich, aber dennoch)2 Unities3 Unities4 Unities5 Unities6 Unities7 Unities8 Unities9 Unities10 Unities Nur für nohlesepiet 10 Unities + Antwort hilfreich

Die Probleme sehe ich nicht im ANSYS, sondern in der durchdachten Aufgabenstellung.
- Wenn zwei Teilchen unterschiedliches Vorzeichen der Ladung und keine Abmessungen haben, was hindert sie, zu einem Teilchen zu verschmelzen?
- Wenn die Teilchen keine Masse haben, was bestimmt dann ihre Geschwindigkeit?

Es ist also zunächst erforderlich, sich ohne Benutzung von ANSYS klar zu machen, was passiert mit zwei geladenen Teilchen?

------------------
Viel Erfolg wünscht
Wolfgang Schätzing

Eine Antwort auf diesen Beitrag verfassen (mit Zitat/Zitat des Beitrags) IP

nohlesepiet
Mitglied



Sehen Sie sich das Profil von nohlesepiet an!   Senden Sie eine Private Message an nohlesepiet  Schreiben Sie einen Gästebucheintrag für nohlesepiet

Beiträge: 18
Registriert: 29.09.2011

erstellt am: 17. Nov. 2011 09:35    Editieren oder löschen Sie diesen Beitrag!  <-- editieren / zitieren -->   Antwort mit Zitat in Fett Antwort mit kursivem Zitat    Unities abgeben: 1 Unity (wenig hilfreich, aber dennoch)2 Unities3 Unities4 Unities5 Unities6 Unities7 Unities8 Unities9 Unities10 Unities

Okay,
wie Sie sicher wissen ist der Betrag der Kraft zwischen zwei Ladungen Q1 und Q2 im Raum zu |K*Q1*Q2/r²|, mit r als dem Abstand zwischen den Teilchen, definiert.
Wenn ich mich nicht irre sollten durch die Singularität im Falle der Koinzidenz der Ladungen ein Verschmelzen verhindert werden? 
Zu den Geschwindigkeiten: Ja ich bekomme unendliche Geschwindigkeiten für den Fall masseloser Teilchen. Da ich an der stationären Endlösung interessiert bin (falls es diese gibt) sollte allerdings der Weg dahin keine Rolle spielen.
Falls diese doch ein Problem darstellt ist es natürlich auch kein Problem (im Sinne des Resultats) eine "Dämpfung" in Form von einer Teilchenmasse vorzusehen. Diese sollte ohne ein überlagertes Gravitationsfeld das Resultat nicht ändern.
Ich hab mir mittlerweile das Buch bzw. das Kapitel mit dem Elektron im Feld besorgt. In meinem Problem erzeugen jedoch die Teilchen das Feld in dem sie sich bewegen selber. Ein Vorgabe der Ladung wie dort, also "händisch" führt nocht zum Erfolg, weil ich noch keine Funktion gefunden habe dem Teilchen eine Ladung zu geben, bzw. ein Potentialfeld um das Teilchen zu erzeugen. 

Eine Antwort auf diesen Beitrag verfassen (mit Zitat/Zitat des Beitrags) IP

nohlesepiet
Mitglied



Sehen Sie sich das Profil von nohlesepiet an!   Senden Sie eine Private Message an nohlesepiet  Schreiben Sie einen Gästebucheintrag für nohlesepiet

Beiträge: 18
Registriert: 29.09.2011

erstellt am: 17. Nov. 2011 10:23    Editieren oder löschen Sie diesen Beitrag!  <-- editieren / zitieren -->   Antwort mit Zitat in Fett Antwort mit kursivem Zitat    Unities abgeben: 1 Unity (wenig hilfreich, aber dennoch)2 Unities3 Unities4 Unities5 Unities6 Unities7 Unities8 Unities9 Unities10 Unities

Zu dem Beispiel in ihrem Buch habe ich noch eine Frage:
1. Warum definieren Sie die Ladung nicht über f,chrg sondern rechnen die Kraftkomponenten selber aus?
2. (Vielleicht ist diese Frage für den Elektrotechniker eine dumme, aber ich bin nun mal keiner) Kann ich einem Punkt gleichzeitig ein Potential (D,VOLT) und eine Ladung (F,CHRG) geben?
Danke

Eine Antwort auf diesen Beitrag verfassen (mit Zitat/Zitat des Beitrags) IP

ChristophN
Mitglied
Berechnungsingenieur


Sehen Sie sich das Profil von ChristophN an!   Senden Sie eine Private Message an ChristophN  Schreiben Sie einen Gästebucheintrag für ChristophN

Beiträge: 773
Registriert: 13.12.2008

erstellt am: 17. Nov. 2011 12:46    Editieren oder löschen Sie diesen Beitrag!  <-- editieren / zitieren -->   Antwort mit Zitat in Fett Antwort mit kursivem Zitat    Unities abgeben: 1 Unity (wenig hilfreich, aber dennoch)2 Unities3 Unities4 Unities5 Unities6 Unities7 Unities8 Unities9 Unities10 Unities Nur für nohlesepiet 10 Unities + Antwort hilfreich

Warum sollte man für solch ein Problem die Finite Elemente Methode verwenden? Es geht doch nur um die Position der Teilchen im Gleichgewichtszustand, oder?

Eine Antwort auf diesen Beitrag verfassen (mit Zitat/Zitat des Beitrags) IP

nohlesepiet
Mitglied



Sehen Sie sich das Profil von nohlesepiet an!   Senden Sie eine Private Message an nohlesepiet  Schreiben Sie einen Gästebucheintrag für nohlesepiet

Beiträge: 18
Registriert: 29.09.2011

erstellt am: 17. Nov. 2011 13:37    Editieren oder löschen Sie diesen Beitrag!  <-- editieren / zitieren -->   Antwort mit Zitat in Fett Antwort mit kursivem Zitat    Unities abgeben: 1 Unity (wenig hilfreich, aber dennoch)2 Unities3 Unities4 Unities5 Unities6 Unities7 Unities8 Unities9 Unities10 Unities

Ja es geht nur um die Teilchenlage im Gleichgewichtszustand. Die FE-Methode verwende ich, bzw will ich verwenden weil es die einfachste Lösung ist die mir eingefallen ist. Ich möchte im Prinzip lediglich den Solver ausnutzen. Ich kann das Problem auch mit Hand ausformulieren, allerdings bekomme ich eine recht große und nicht besonders schöne Matrizengleichung die dann (da sie nichtlinear ist) iterativ gelöst werden muss. Zumal die Formulierung der periodischen Randbedingungen sehr aufwendig ist.
Ich dachte, dass das ganze mehr oder weniger ein Standartproblem der Elektrotechnik ist und daher der Aufwand für eine Simulation in Ansys wesentlich geringer ist also das ganze z.B. in Matlab zu implementieren.
Allerdings entzieht sich momentan die Ansys Simulation meinen zugegebenermaßen beschränkten Kenntnissen auf dem Gebiet der Elektrotechnik.
Mein Ansatz lautet soweit:
Ich platziere Knoten (ich habe mass21 Elemente verwendet bisher, aber diese haben anscheinend den notwendigen elektrischen Freiheitsgrad nicht) ohne mechanische Randbedingungen in einem Rechteck aus Solid121 Elementen. Den mass21 Elementen (teilchen) weise ich mit F,i,CHRG,Ladung eine Ladung zu. Wahlweise können die Elemente auch eine Masse zugewiesen bekommen, falls die unendliche theoretische Geschwindigkeit Probleme verursacht. Am Rand meines Rechtecks gebe ich die Randbedingungen mittels Constraint Equations vor. Da die Zelle periodisch sein soll muss die Intensität des elektrischen Feldes am linken und rechten Rand sowie am oberen und unteren Rand gleich sein. Ich habe bisher noch keine Möglichkeit gefunden die Feldstärke explizit vorzugeben daher habe ich bisher mit dem Potential, welches über D,i,VOLT definiert werden kann (oder im Falle der CE über CE,#,,,VOLT....)gearbeitet. Das elektrische Potential phi und die Feldstärke E sind über E=grad(phi) gekoppelt.
Mit diesem Ansatz bekomme ich allerdings weder ein Feld (welches die Ladungen auf den mass21 Teilchen erzeugen sollten) noch sonst irgendwas sondern einfach kein Feld, keine Kräfte....
Falls Sie eine bessere Idee haben bin ich natürlich nicht abgeneigt...
Danke und Grüße

[Diese Nachricht wurde von nohlesepiet am 17. Nov. 2011 editiert.]

Eine Antwort auf diesen Beitrag verfassen (mit Zitat/Zitat des Beitrags) IP

nohlesepiet
Mitglied



Sehen Sie sich das Profil von nohlesepiet an!   Senden Sie eine Private Message an nohlesepiet  Schreiben Sie einen Gästebucheintrag für nohlesepiet

Beiträge: 18
Registriert: 29.09.2011

erstellt am: 17. Nov. 2011 17:22    Editieren oder löschen Sie diesen Beitrag!  <-- editieren / zitieren -->   Antwort mit Zitat in Fett Antwort mit kursivem Zitat    Unities abgeben: 1 Unity (wenig hilfreich, aber dennoch)2 Unities3 Unities4 Unities5 Unities6 Unities7 Unities8 Unities9 Unities10 Unities

Ach so Herr Schätzing ich habe mich schon gewundert. Sie haben sich wohl verlesen:
 
Zitat:
gleichem Vorzeichen jedoch unterschiedlicher Ladung

(Beitrag von gestern Abend)
gemeint ist unterschiedliche Ladung |Qi| der Teilchen mit gleichem Vorzeichen für alle.

[Diese Nachricht wurde von nohlesepiet am 17. Nov. 2011 editiert.]

Eine Antwort auf diesen Beitrag verfassen (mit Zitat/Zitat des Beitrags) IP

wosch
Ehrenmitglied V.I.P. h.c.
Elektrotechniker im Ruhestand


Sehen Sie sich das Profil von wosch an!   Senden Sie eine Private Message an wosch  Schreiben Sie einen Gästebucheintrag für wosch

Beiträge: 2607
Registriert: 16.12.2004

Rechne zuerst ein Problem nach, für das Du eine analytische Lösung kennst.

erstellt am: 17. Nov. 2011 17:43    Editieren oder löschen Sie diesen Beitrag!  <-- editieren / zitieren -->   Antwort mit Zitat in Fett Antwort mit kursivem Zitat    Unities abgeben: 1 Unity (wenig hilfreich, aber dennoch)2 Unities3 Unities4 Unities5 Unities6 Unities7 Unities8 Unities9 Unities10 Unities Nur für nohlesepiet 10 Unities + Antwort hilfreich

Wenn die Teilchen alle Ladungen gleichen Vorzeichens haben und sich in einem homogenen elektrostatischen Feld befinden, wird die Kraft sie alle in die gleiche Richtung beschleunigen, woher soll dann ein Gleichgewichtszustand kommen?
(Kommen die Teilchen an eine Elektrode, geben sie ihre Ladung ab.)
Wenn kein äußeres Feld anliegt, werden alle Teilchen ins Unendliche auseinander streben. Wer soll sie aufhalten?

------------------
Viel Erfolg wünscht
Wolfgang Schätzing

Eine Antwort auf diesen Beitrag verfassen (mit Zitat/Zitat des Beitrags) IP

nohlesepiet
Mitglied



Sehen Sie sich das Profil von nohlesepiet an!   Senden Sie eine Private Message an nohlesepiet  Schreiben Sie einen Gästebucheintrag für nohlesepiet

Beiträge: 18
Registriert: 29.09.2011

erstellt am: 17. Nov. 2011 18:50    Editieren oder löschen Sie diesen Beitrag!  <-- editieren / zitieren -->   Antwort mit Zitat in Fett Antwort mit kursivem Zitat    Unities abgeben: 1 Unity (wenig hilfreich, aber dennoch)2 Unities3 Unities4 Unities5 Unities6 Unities7 Unities8 Unities9 Unities10 Unities


Bild1.png

 

Es soll kein äußeres Feld anliegen, derjenige der in diesem Falle meine Teilchen davon abhält ins unendliche zu streben ist die Periodizität.
Die ist, zugegebenermaßen recht rudimentär, in den Bildern angedeutet. Einfach wäre vielleicht die Vorstellung zunächst einmal auf eine Dimension (Linie) zu beschränken, da anscheinend ein Kommunikationsproblem vorliegt:

Wir betrachten dafür einen Abschnitt der Länge L mit N Teilchen positiver Ladung mit verschiedener Intensität. Dieser Abschnitt wiederholt sich dabei nach links und nach rechts unendlich oft. (Der Abschnitt der Länge L ist also repäsentativ für das Problem)
Es ist also analog, sich einen Kreis vorzustellen auf dem sich die N Teilchen bewegen können (Es ist KEINE radiale Bewegung möglich).
Da die Teilchen unterschiedliche Ladung haben werden sie sich i.A. nicht äquidistant auf dem Kreis verteilen sondern eine andere Konfiguration einnehmen. Diese Konfiguration suche ich und zwar im 2D Fall.
Ich habe der Klarheit halber eine Schemazeichnung angehängt.
Wie setze ich diese Problemstellung in ANSYS um?

Eine Antwort auf diesen Beitrag verfassen (mit Zitat/Zitat des Beitrags) IP

Anzeige.:

Anzeige: (Infos zum Werbeplatz >>)

Darstellung des Themas zum Ausdrucken. Bitte dann die Druckfunktion des Browsers verwenden. | Suche nach Beiträgen

nächster neuerer Beitrag | nächster älterer Beitrag
Antwort erstellen


Diesen Beitrag mit Lesezeichen versehen ... | Nach anderen Beiträgen suchen | CAD.de-Newsletter

Administrative Optionen: Beitrag schliessen | Archivieren/Bewegen | Beitrag melden!

Fragen und Anregungen: Kritik-Forum | Neues aus der Community: Community-Forum

(c)2023 CAD.de | Impressum | Datenschutz